Last visit was: 26 Apr 2024, 03:28 It is currently 26 Apr 2024, 03:28

Close
GMAT Club Daily Prep
Thank you for using the timer - this advanced tool can estimate your performance and suggest more practice questions. We have subscribed you to Daily Prep Questions via email.

Customized
for You

we will pick new questions that match your level based on your Timer History

Track
Your Progress

every week, we’ll send you an estimated GMAT score based on your performance

Practice
Pays

we will pick new questions that match your level based on your Timer History
Not interested in getting valuable practice questions and articles delivered to your email? No problem, unsubscribe here.
Close
Request Expert Reply
Confirm Cancel
SORT BY:
Date
Tags:
Show Tags
Hide Tags
Intern
Intern
Joined: 18 Jul 2019
Posts: 36
Own Kudos [?]: 35 [23]
Given Kudos: 128
Send PM
Most Helpful Reply
Volunteer Expert
Joined: 16 May 2019
Posts: 3512
Own Kudos [?]: 6860 [10]
Given Kudos: 500
General Discussion
Current Student
Joined: 25 Oct 2018
Posts: 16
Own Kudos [?]: 4 [0]
Given Kudos: 33
Location: India
Concentration: Finance, Leadership
GMAT 1: 740 Q50 V41
GPA: 3.37
WE:Engineering (Health Care)
Send PM
Intern
Intern
Joined: 22 Feb 2020
Posts: 2
Own Kudos [?]: 0 [0]
Given Kudos: 11
Location: India
GMAT 1: 660 Q49 V32
Send PM
Re: If the price it pays for coffee beans continues to increase, the Coffe [#permalink]
Could you pease help me understand this question with proper solution?
Senior Manager
Senior Manager
Joined: 24 Nov 2019
Posts: 284
Own Kudos [?]: 263 [2]
Given Kudos: 811
Location: Bangladesh
GMAT 1: 590 Q44 V27
GMAT 2: 600 Q46 V27
GMAT 3: 690 Q47 V37
GPA: 3.5
Send PM
Re: If the price it pays for coffee beans continues to increase, the Coffe [#permalink]
2
Kudos
(A) If the Coffee Shoppe's overall profitability decreases, the price it pays for coffee beans will have continued to increase.(No, if the price it pays for coffee beans has continued to increase, then the Coffee Shoppe's overall profitability will decrease)

(B) If the Coffee Shoppe's overall profitability decreases, either it will have begun selling noncoffee products or its coffee sales will have decreased.( No, selling noncoffee products or decreasing sales have negative effect on overall profitability)

(C) The Coffee Shoppe's overall profitability will decrease if the price it pays for coffee beans continues to increase. Yes


(D) The price it pays for coffee beans cannot decrease without the Coffee Shoppe's overall profitability also decreasing. (No, actually the Coffee Shoppe's overall profitability can not decrease without the price it pays for coffee beans increases)

(E) Either the price it pays for coffee beans will continue to increase or the Coffee Shoppe's coffee sales will increase ( They can happen at the same time)
Intern
Intern
Joined: 04 Apr 2020
Posts: 43
Own Kudos [?]: 12 [0]
Given Kudos: 14
Send PM
Re: If the price it pays for coffee beans continues to increase, the Coffe [#permalink]
I think (C) is literally mentioned in the argument
Manager
Manager
Joined: 08 Dec 2020
Posts: 62
Own Kudos [?]: 33 [0]
Given Kudos: 923
Send PM
Re: If the price it pays for coffee beans continues to increase, the Coffe [#permalink]
AndrewN: I could eliminate all but options (B) & (C). I understood why option (B) is incorrect. But I have a doubt regarding option (C). It was because of this doubt that I omitted option (C) and marked option (B).

My doubt is: The Coffee Shoppe's overall profitability will decrease if the price it pays for coffee beans continues to increase.: If the price it pays for coffee beans continues to increase & further if Coffee Shoppe cascade this hike to their coffee products & if the sales of the coffee products don't go down, then, in that case, Coffee Shoppe's profitability will not go down.

Is my inference correct? In the above case, there is no guarantee that profitability will go down for sure even if the prices of the coffee beans go up. Kindly clarify?

Your response is awaited. Regards.
Volunteer Expert
Joined: 16 May 2019
Posts: 3512
Own Kudos [?]: 6860 [1]
Given Kudos: 500
Re: If the price it pays for coffee beans continues to increase, the Coffe [#permalink]
1
Kudos
Expert Reply
beeblebrox wrote:
AndrewN: I could eliminate all but options (B) & (C). I understood why option (B) is incorrect. But I have a doubt regarding option (C). It was because of this doubt that I omitted option (C) and marked option (B).

My doubt is: The Coffee Shoppe's overall profitability will decrease if the price it pays for coffee beans continues to increase.: If the price it pays for coffee beans continues to increase & further if Coffee Shoppe cascade this hike to their coffee products & if the sales of the coffee products don't go down, then, in that case, Coffee Shoppe's profitability will not go down.

Is my inference correct? In the above case, there is no guarantee that profitability will go down for sure even if the prices of the coffee beans go up. Kindly clarify?

Your response is awaited. Regards.

Hello, beeblebrox. I understand your real-world chain of logic here. But remember, both the GMAT™ and the LSAT employ airtight logic in their own bubbles, and we have to appreciate the vacuum. In that light, I have taken the liberty of highlighting the wayward direction within your thought process. The passage, within the first two lines, categorically dismisses the possibility that the coffee shop can both pay higher prices for coffee beans and continue selling as much coffee. It is an either/or proposition instead. (My bold italics.)

Quote:
If the price it pays for coffee beans continues to increase, the Coffee Shoppe will have to increase its prices. In that case, either the Coffee Shoppe will begin selling noncoffee products or its coffee sales will decrease.

In this situation, we know the coffee shop will enter a downward spiral: sell noncoffee products at a loss, or sell less coffee, all based on the rising price of coffee beans.

One more thought on CR in general. It is better to get behind an answer you think is safer than to go with one you can definitely pin a doubt to. I know that sinking feeling of chasing an answer simply because I was unsure about something or other in another answer choice, only to find out my original answer was correct. Put into practice a firmer method, though, something you can apply to question after question, and accuracy will follow, even on these tough ones.

I hope that helps clarify the matter. Thank you for thinking to ask.

- Andrew
User avatar
Intern
Intern
Joined: 16 Nov 2022
Posts: 1
Own Kudos [?]: 1 [1]
Given Kudos: 0
Send PM
If the price it pays for coffee beans continues to increase, the Coffe [#permalink]
1
Kudos
First premise. I denoted it as, P-->Q. "If price it pays for coffee beans increase then the shop will increase its price"

Second premise denoted as, Q-->(R v S).
"In case Q---the shop increasing its price----, then either R---shop will begin selling non-coffee products---or S--coffee sales will decrease."

Third premise. R-->T. "If shop sells non-coffee products then overall profitability will decrease."

Fourth premise. ~T-->~S. "shop can avoid decreasing profit only if its coffee sales do not decrease."

the ~ denotes a negation and key word, "only if" is the consequent of a conditional.

Just a quick recap on conditionals: there are 2 components, the antecedent and the consequent.
the antecedent of a conditional is the "if" part of a conditional statement for example, "IF I go for a walk today, THEN it is sunny"
and the term, "only if" is equivalent to "then" thus, the fourth premise is symbolically denoted as ~T-->~S.


Now let's take a look at each conclusion and see which ones are valid.

A) If overall profitability decreases, the price it pays for coffee beans will have continued to increase.

This is symbolically equivalent to "If T then P".

Right away I can tell this is incorrect based on truth tables for conditional logic.

P --> Q
T T T
T F F
F T T
F T F

So the T's and F's represent true or false; it is binary and for a conditional, there are 4 total possible combinations of the validity for statement P and statement Q.
I will not go into detail on truth tables, you can wikipedia it yourself to follow along.
We see that in a conditional, there is only one instance that would make a conditional statement false, which is when the antecedent is true and the consequent
is false.
It basically means that when you have a conditional if the antecedent is true, then the consequent must necessarily follow.
However, we see that the consequent can be True even if the antecedent is false.
Where validity of P is False and validity for Q is true.

Now for conclusion A, we see that in order for T--"profit decrease" to even be valid, it requires the antecedent R---sells noncoffee products. And R requires the antecedent Q---increase sale of coffee beans.

Q. Q ---> (R v S). R-->T.

And as we've seen, Q can occur even if P does not occur, so P---price it pays for coffee beans increase---can never be contingent on T---profit decrease.


For conclusin B) If the shops overall profitability decreases, either it will have begun selling non-coffee products or its cotton sales will have decreased.

T --> (R v S).

here we have the same problem as A. the antecedent and consequents are flipped and they are not equivalent. the only way a conditional is equal both ways is if it is a bi-conditional, the key term for bi-conditional is "if and only if"


conclusion C) the shops overall profit will decrease if the price it pays for coffee beans continues to increase.

P --> T.

Another way to word the conclusion is, "if the price it pays for coffee beans continue to increase, the shops overall profit will decrease".

So here, I will show you what is called a derivation. Basically its like accounting but for arguments.
Premises: P--->Q. Q-->(R v S). R-->T. ~T-->~S.
Conclusion: P-->T
[align=]
1) Prove P-->T -----------------> line 3, conditional proof

2) P Assume the conditional

3) Prove T -----------------> line 9, direct proof

4) ~T Assume the indirect

5) ~S line 4, Premise 4. Modus Ponens

6) Q line 2, Premise 1. Modus Ponens

7) R v S line 6, Premise 2. Modus Ponens

8) R line 5, line 7, Modus Tollendo Ponens (disjunctive syllogism)

9) T line 8, Premise 3. Modus Ponens


[/align]

Now on line 3 we can provide our reasoning which is a direct proof of the consequent T.
And finally, for line 1, we proved the conditional P-->T


logically speaking, conclusion C is valid, given the premises, therefore C is the correct answer.


Edit:

Sorry I do not know how to align properly in this text editor. i will post a screenshot of the derivation using a logic software.
Manager
Manager
Joined: 26 Dec 2022
Posts: 89
Own Kudos [?]: 21 [1]
Given Kudos: 46
Location: India
GMAT 1: 710 Q50 V36
Send PM
If the price it pays for coffee beans continues to increase, the Coffe [#permalink]
1
Kudos
(A) If the Coffee Shoppe's overall profitability decreases, the price it pays for coffee beans will have continued to increase. [ We can't say that---we don't know what will happen when Price of coffee beans decreases. It also might lead to decrease in overall profitability]
(B) If the Coffee Shoppe's overall profitability decreases, either it will have begun selling noncoffee products or its coffee sales will have decreased. [ Same reason: We can't say that---we don't know what will happen when Price of coffee beans decreases. It also might lead to decrease in overall profitability]
(C) The Coffee Shoppe's overall profitability will decrease if the price it pays for coffee beans continues to increase. [Yes! look at the tree diagram; we can infer this one]
(D) The price it pays for coffee beans cannot decrease without the Coffee Shoppe's overall profitability also decreasing.
(E) Either the price it pays for coffee beans will continue to increase or the Coffee Shoppe's coffee sales will increase.

Below is the tree diagram, which you might find helpful. 
Attachments

Capture.JPG
Capture.JPG [ 51.31 KiB | Viewed 1214 times ]

GMAT Club Bot
If the price it pays for coffee beans continues to increase, the Coffe [#permalink]
Moderators:
GMAT Club Verbal Expert
6921 posts
GMAT Club Verbal Expert
238 posts
CR Forum Moderator
832 posts

Powered by phpBB © phpBB Group | Emoji artwork provided by EmojiOne